Proving this series converges or diverges

  • Thread starter bonfire09
  • Start date
  • Tags
    Series
In summary: Of course in this case, the limit is -ln(2).In summary, the conversation discusses the convergence or divergence of the sequence y_n=\frac{1}{n+1}+\frac{1}{n+2}+...+\frac{1}{2n} where n\in\mathbb{N}. The conversation goes through various attempts at proving its divergence, including using partial sums and comparing it to other series. Eventually, it is shown that the sequence is monotone increasing and bounded above, leading to the conclusion that it converges to the limit -ln(2).
  • #1
bonfire09
249
0

Homework Statement


Prove [itex] y_n=\frac{1}{n+1}+\frac{1}{n+2}+...+\frac{1}{2n} [/itex] where [itex]n\in\mathbb{N}[/itex] is convergent or divergent.

Homework Equations


The Attempt at a Solution


I am stuck on this. I think its divergent but I am having trouble proving that it is divergent. I tried to break up the sequence using partial sums but I don't see a pattern. What I also don't get is what [itex]y_1[/itex] is equal too. Is it [itex]y_1=\frac{1}{1+1}+\frac{1}{1+2}+...+\frac{1}{2*1} [/itex] or is it equal to [itex]y_1=\frac{1}{1+1} [/itex]? I've been stuck for quite a while.
 
Last edited:
Physics news on Phys.org
  • #2
bonfire09 said:

Homework Statement


Prove [itex] y_n=\frac{1}{n+1}+\frac{1}{n+2}+...+\frac{1}{2n} [/itex] where [itex]n\in\mathbb{N}[/itex] is convergent or divergent.


Homework Equations





The Attempt at a Solution


I am stuck on this. I think its divergent but I am having trouble proving that it is divergent. I tried to break up the sequence using partial sums but I don't see a pattern. What I also don't get is what [itex]y_1[/itex] is equal too. Is it [itex]y_1=\frac{1}{1+1}+\frac{1}{1+2}+...+\frac{1}{2*1} [/itex] or is it equal to [itex]y_1=\frac{1}{1+1} [/itex]? I've been stuck for quite a while.
From what I understand, it looks like this:
$$y_n=\sum_{k=n+1}^{2n}\frac{1}{k}.$$
That help?
 
  • #3
Yes but how would I go about proving it is divergent. I was thinking of using [itex] \frac{1}{n^2}+...+\frac{1}{n^2}[/itex] since [itex] \frac{1}{n+1}+...+\frac{1}{2n}>\frac{1}{n^2}+...+\frac{1}{n^2} [/itex]?{Edit*I don't think it works}. I was thinking of using [itex] \frac{1}{n+1}+...+\frac{1}{n+1}[/itex]. I don't think the way you wrote the series works. I think the limits have to be adjusted.
 
Last edited:
  • #4
No, that doesn't work. Since the denominators are all "of order n", it would be better to compare the terms to 1/n (or your suggestion 1/(n+1)). For all i between 1 and n, [itex]1/(n+i)\ge 1/(2n)[/itex]
 
  • #5
bonfire09 said:
Yes but how would I go about proving it is divergent. I was thinking of using [itex] \frac{1}{n^2}+...+\frac{1}{n^2}[/itex] since [itex] \frac{1}{n+1}+...+\frac{1}{2n}>\frac{1}{n^2}+...+\frac{1}{n^2} [/itex]?{Edit*I don't think it works}. I was thinking of using [itex] \frac{1}{n+1}+...+\frac{1}{n+1}[/itex]. I don't think the way you wrote the series works. I think the limits have to be adjusted.
I...don't think so. Check again. I'm pretty sure my expression for the summation is correct.

I'd use a test of convergence. Are you familiar with any?
 
  • #6
Well this what I get so far that [itex] \sum_{k=n+1}^{2n}\frac{1}{k}≥ \sum_{k=n+1}^{2n}\frac{1}{k+1} [/itex] Hence its divergent. I think it works since [itex] \sum_{k=n+1}^{2n}\frac{1}{k+1}=\frac{1}{(n+1)+1}+\frac{1}{(n+2)+1}+...+\frac{1}{2n+1}[/itex] ≤ [itex] \sum_{k=n+1}^{2n}\frac{1}{k}= \frac{1}{n+1}+\frac{1}{n+2}+...+\frac{1}{2n} [/itex]
 
Last edited:
  • #7
This is a little bit subtle

$$\frac{1}{2}=\frac{n}{2n}<\sum_{k=1}^n \frac{1}{n+k}<\frac{n}{n+1}<1$$

hint:think log(n)

hint2:
if

$$H_n=\sum_{k=1}^n \frac{1}{k}$$

then

$$\sum_{k=1}^n \frac{1}{n+k}=H_{2n}-H_{n}$$
 
Last edited:
  • #8
so is the way I have it not correct then?
 
  • #9
^The way you wrote the sum is right, but you cannot conclude it diverges from what you have done.
 
  • #10
bonfire09 said:
so is the way I have it not correct then?

If you know what a Riemann sum is, write one for the integral of the function 1/x from x=1 to x=2.
 
  • #11
Ok I will try other methods. I would think there would be a much simpler way of doing this because Bartle (book I am using) presents convergence and divergence of series before he introduces integration and differentiation.
 
  • #12
bonfire09 said:
Ok I will try other methods. I would think there would be a much simpler way of doing this because Bartle (book I am using) presents convergence and divergence of series before he introduces integration and differentiation.

Fair enough. You can also show a sequence converges if you can show it's monotone and bounded. Can you show ##y_{n+1}-y_{n}## is positive so it's monotone increasing and, as lurflurf indicated, bounded above?
 
  • #13
So [itex]y_{n+1}= \frac{1}{n+2}+\frac{1}{n+3}+...+\frac{1}{2n+1}[/itex] and [itex] y_n=\frac{1}{n+1}+\frac{1}{n+2}+...+\frac{1}{2n}[/itex]. So [itex] y_{n+1}-y_n= \frac{-1}{n+1}+\frac{1}{2n+1}[/itex]. I think that is how it supposed to go.
 
  • #14
bonfire09 said:
So [itex]y_{n+1}= \frac{1}{n+2}+\frac{1}{n+3}+...+\frac{1}{2n+1}[/itex] and [itex] y_n=\frac{1}{n+1}+\frac{1}{n+2}+...+\frac{1}{2n}[/itex]. So [itex] y_{n+1}-y_n= \frac{-1}{n+1}+\frac{1}{2n+1}[/itex]. I think that is how it supposed to go.

Right idea, but the last term in ##y_{n+1}## is ##\frac{1}{2(n+1)}##, not ##\frac{1}{2n+1}##. Do it a little more carefully. It might be good to write out the n=1, n=2, n=3 cases explicitly, then do some differences and compare with what you think the answer is symbolically. That's what I do.
 
Last edited:
  • #15
Oh yeah I forgot. [itex]y_{n+1}= \frac{1}{n+2}+\frac{1}{n+3}+...+\frac{1}{2n+5}[/itex] and [itex] y_n=\frac{1}{n+1}+\frac{1}{n+2}+...+\frac{1}{2n}[/itex] So [itex] y_{n+1}-y_n=\frac{-1}{n+1}+\frac{1}{2n+1}+\frac{1}{2n+2}[/itex] which seems to be decreasing that is [itex] y_{n+1}\leq y_n[/itex]
 
  • #16
bonfire09 said:
Oh yeah I forgot. [itex]y_{n+1}= \frac{1}{n+2}+\frac{1}{n+3}+...+\frac{1}{2n+5}[/itex] and [itex] y_n=\frac{1}{n+1}+\frac{1}{n+2}+...+\frac{1}{2n}[/itex] So [itex] y_{n+1}-y_n=\frac{-1}{n+1}+\frac{1}{2n+1}+\frac{1}{2n+2}[/itex] which seems to be decreasing.

Ok, so why do you think [itex]\frac{-1}{n+1}+\frac{1}{2n+1}+\frac{1}{2n+2}[/itex] is negative? I don't. Just realized 2n+5 has got to be a typo, since your difference is correct.
 
Last edited:
  • #17
Oh yes its increasing. Now once I show that [itex] \frac{-1}{n+1} \leq \frac{1}{2n+1}+\frac{1}{2n+2}[/itex]. Now I think I'm going to have to use a epsilon proof to show [itex] y_n[/itex] its convergent which I need some help with.
 
  • #18
bonfire09 said:
Oh yes its increasing. Now once I show that [itex] \frac{-1}{n+1} \leq \frac{1}{2n+1}+\frac{1}{2n+2}[/itex]. Now I think I'm going to have to use a epsilon proof to show its convergent.

That's a little garbled too. My whole point is that there is likely a theorem in Bartle that if a sequence is monotone increasing and bounded above, then it converges. Use the theorem. No need for epsilons if you've got the theorem.
 
Last edited:
  • #19
It says 3.3.2 Monotone Convergence Theorem A monotone, sequence of real numbers is
convergent if and only if it is bounded. Further: (a) If X = (xn) is a bounded increasing sequence, then [itex] lim(x_n) = sup\{x_n : n ε N\} [/itex]. I some how have to show [itex]y_n[/itex] is bounded and all I know its increasing so it is monotone.
 
  • #20
bonfire09 said:
It says 3.3.2 Monotone Convergence Theorem A monotone, sequence of real numbers is
convergent if and only if it is bounded. Further: (a) If X = (xn) is a bounded increasing sequence, then [itex] lim(x_n) = sup\{x_n : n ε N\} [/itex]. I some how have to show [itex]y_n[/itex] is bounded and all I know its increasing so it is monotone.

lurflurf already hinted you that it's bounded. You are summing n numbers and they are all less than 1/n, so? Doesn't that mean it's bounded? Are you clear on the monotone bit? Why is ##\frac{-1}{n+1}+\frac{1}{2n+1}+\frac{1}{2n+2}## positive?
 
  • #21
Well [itex] \frac{-1}{n+1}<\frac{1}{2n+1}+\frac{1}{2n+2}[/itex] since if we start from [itex] \frac{-3}{n+1}<\frac{2}{2n+1}=\frac{-2}{n+1}<\frac{2}{2n+1}+\frac{1}{n+1}=\frac{-2}{n+1}<\frac{2}{2n+1}+\frac{2}{2n+2}=\frac{-1}{n+1}<\frac{1}{2n+1}+\frac{1}{2n+2} [/itex] Thus its positive. Oh yeah I saw the hint. Since its bounded and monotone its convergent by the monotone convergent theorem.
 
  • #22
##\frac{-1}{n+1}<\frac{1}{2n+1}+\frac{1}{2n+2}## is certainly true. But that's useless. You want to prove ##\frac{-1}{n+1}+\frac{1}{2n+1}+\frac{1}{2n+2} \gt 0##. That's completely different. Do the algebra and combine the fractions.
 
  • #23
Well [itex] \frac{1}{n+1}<\frac{1}{2n+1}+\frac{1}{2n+2}[/itex] since if we start from [itex] \frac{1}{2n+2}<\frac{1}{n+1}=\frac{2}{2n+2}<\frac{1}{2n+1}+\frac{1}{2n+2}→\frac{1}{2n+1}+\frac{1}{2n+2}-\frac{1}{n+1}>0 [/itex] Thus its positive. Oh yeah I saw the hint. Since its bounded and monotone its convergent by the monotone convergent theorem.
 
  • #24
bonfire09 said:
Well [itex] \frac{1}{n+1}<\frac{1}{2n+1}+\frac{1}{2n+2}[/itex] since if we start from [itex] \frac{1}{2n+2}<\frac{1}{n+1}=\frac{2}{2n+2}<\frac{1}{2n+1}+\frac{1}{2n+2}→\frac{1}{2n+1}+\frac{1}{2n+2}-\frac{1}{n+1}>0 [/itex] Thus its positive. Oh yeah I saw the hint. Since its bounded and monotone its convergent by the monotone convergent theorem.

Ok, I find that a little hard to follow, but it's probably ok. Yes, ##y_n## is increasing and bounded above. So it converges.
 
  • Like
Likes 1 person
  • #25
Thanks so much. I think I am going to practice on other series and sequences until I get a good hang of it. I don't think its that bad now.
 
  • #26
Just a note, the value the sequence converges to can be found this way:

[tex]\lim_{n\to\infty} \sum_{k=0}^{2n} \frac{1}{k} - \sum_{k=0}^{n} \frac{1}{k}
= \lim_{n\to\infty} \sum_{k=0}^{2n} \frac{1}{k} - \log(2n) - \sum_{k=0}^{n} \frac{1}{k} + \log(n) + \log(2)
= \gamma - \gamma + \log(2) = \log(2)[/tex]
 
  • #27
Millennial said:
Just a note, the value the sequence converges to can be found this way:

[tex]\lim_{n\to\infty} \sum_{k=0}^{2n} \frac{1}{k} - \sum_{k=0}^{n} \frac{1}{k}
= \lim_{n\to\infty} \sum_{k=0}^{2n} \frac{1}{k} - \log(2n) - \sum_{k=0}^{n} \frac{1}{k} + \log(n) + \log(2)
= \gamma - \gamma + \log(2) = \log(2)[/tex]

Yeah, that's what lurflurf was pointing out. I think it's more elementary to approach it as a Riemann sum as in post 10.
 

1. What does it mean for a series to converge or diverge?

When we say that a series converges, it means that the terms in the series approach a finite limit as the number of terms in the series increases. In other words, the series gets closer and closer to a specific value. On the other hand, if a series diverges, it means that the terms in the series do not approach a finite limit and the series does not have a specific value.

2. How can I prove that a series converges or diverges?

To prove whether a series converges or diverges, we need to use various convergence tests such as the limit comparison test, ratio test, or integral test. These tests help us determine the behavior of the series and whether it approaches a finite limit or not.

3. Can a series converge and diverge at the same time?

No, a series can either converge or diverge, but not both at the same time. If a series approaches a finite limit, it converges, and if it does not approach a finite limit, it diverges.

4. What happens if the convergence tests are inconclusive?

If the convergence tests are inconclusive, it means that we cannot determine whether the series converges or diverges using those specific tests. In this case, we may need to use other methods such as the alternating series test or the root test to determine the convergence or divergence of the series.

5. Can you provide an example of a series that converges and another that diverges?

An example of a convergent series is the geometric series, where the ratio between consecutive terms approaches a finite limit. On the other hand, an example of a divergent series is the harmonic series, where the sum of the terms increases without bound.

Similar threads

  • Calculus and Beyond Homework Help
Replies
3
Views
401
  • Calculus and Beyond Homework Help
Replies
2
Views
169
  • Calculus and Beyond Homework Help
Replies
1
Views
231
  • Calculus and Beyond Homework Help
Replies
2
Views
703
  • Calculus and Beyond Homework Help
Replies
10
Views
465
  • Calculus and Beyond Homework Help
Replies
4
Views
281
  • Calculus and Beyond Homework Help
Replies
17
Views
592
  • Calculus and Beyond Homework Help
Replies
14
Views
1K
  • Calculus and Beyond Homework Help
Replies
5
Views
475
  • Calculus and Beyond Homework Help
Replies
17
Views
1K
Back
Top